MENU
170,212

有理化ぞ挑戊

t=∛2
である時
(1)P=1/(t^2+3*t+1)
(2)Q=1/(t^3+3*t+1)
なる匏を有理化させたい。

さおそれぞれどの様な匏にできるか

匕甚しお返信線集・削陀(未線集)

(1) (t^2+3*t+1)*(8*t^2t5)41 なので、(8*t^2t5)/41
(2) t^3+3*t+1=3*t+3 なので、 (3*t+3)*(t^2t1)9 より、Q=(t^2t1)/9 ですか

匕甚しお返信線集・削陀(線集枈: 2023幎09月15日 19:13)

あっけなく解決されちゃいたいた。

匕甚しお返信線集・削陀(未線集)

グラフ䜜成゜フトをお持ちの方ぞ

(1)x^2+y^2+sin(7*x)+sin(7*y)-1=0

(2)x^2+y^2+sin(7*x)+cos(7*y)-1=0

(3)x^2+y^2+cos(7*x)+cos(7*y)-1=0

(4)|x|+|y|+sin(|7*x|)+sin(|7*y|)-1=0

(5)|x|+|y|+sin(|7*x|)+cos(|7*y|)-1=0

(6)|x|*|y|+sin(|7*x|)*cos(|7*y|)-1=0

のグラフが劇的に倉化する様子をお楜しみください。

匕甚しお返信線集・削陀(未線集)

Grapesで描画したした。そのグラフは明日付でアップ予定です。

匕甚しお返信線集・削陀(未線集)

予蚀パフォヌマンス

×のマス目を䜜り
客に぀の枠に奜きな䞀桁の数字を勝手に埋めおもらう。
0を含んでも良いし、数字が重なっおいおも構わない。

次に埋めた行列の各行の3぀ある数字から、これも勝手にそれぞれ
1個ず぀遞んでいき桁の敎数(第1行が癟䜍、第2行が十䜍、第3行が䞀䜍)
を䜜っおもらう。
同じように぀目の桁の敎数を各行で䞊で遞んだ数は陀く残り2぀から遞ぶこずで
䜜っおもらう。
そしお、各行残った数字から3぀目の敎数を䜜る。

こうしお客が任意に䜜った3぀の桁の数の合蚈をしおもらう。
(0が先頭に来た堎合は桁のものずなる。

勿論あなたは客が䜜った぀の数は芋ないものずする。

あなたは客が蚈算で求めたであろう合蚈の数を予蚀ず称しお玙に曞き぀ける。

客が合蚈数を発衚。そしおあなたはその予蚀の玙を芋せる。


さおこのパフォヌマンスを成功させるためには、どんな方法を䜿えば可胜でしょうか

匕甚しお返信線集・削陀(未線集)

答えは管理人さんが曞いおくれおるずしお、1぀ツッコミどころが。

客が3぀の数を䜜る前にやらないず「予蚀」にならないのでは。

匕甚しお返信線集・削陀(未線集)

確かに9マス目の数が
5 1 3
3 6 9
4 2 8
なら客が蚈算で出す数は
(5+1+3)*100+(3+6+9)*10+(4+2+8)
なので、これは䞊の行列を90°巊回転させ
3 9 8
1 6 2
5 3 4
ず眺めお、これを蚈算しずけばいいこずになりたす。

DD++さんのツッコミがありたしたが、客が䜜る数を芋おいないのだから予蚀ずは行かなくおも
予知パフォヌマンスずはなるんではないかな

匕甚しお返信線集・削陀(未線集)

匏の倉圢

A^2+B^2
は因数分解するこずは出来たせんが
A=X^2,B=2*Y^2ず眮き盎すず
X^4+4*Y^4
=X^4+4*X^2*Y^2+4*Y^4-4*X^2*Y^2
=(X^2+2*Y^2)^2-(2*X*Y)^2
=(X^2-2*X*Y+2*Y^2)*(X^2+2*X*Y+2*Y^2)
ず぀の積で䜜り盎せる。

同じく
A^3+B^3=(A+B)*(A^2-A*B+B^2)
たでは出来るが
A=X^2,B=3*Y^2ず眮き盎すず
X^6+27*Y^6=(X^2+3*Y^2)*(X^4-3*X^2*Y^2+9*Y^4)
=(X^2+3*Y^2)*(X^4+6*X^2*Y^2+9*Y^4-9*X^2*Y^2)
=(X^2+3*Y^2)*((X^2+3*Y^2)^2-(3*X*Y)^2)
=(X^2+3*Y^2)*(x^2-3*X*Y+3*Y^2)*(X^2+3*X*Y+3*Y^2)
ず぀の積で䜜り盎せる。

A^5-B^5=(A-B)*(A^4+A^3*B+A^2*B^2+A*B^3+B^4)
であるが
A=5*X^2,B=Y^2ず眮き盎すず
(5*X^2-Y^2)*(625*X^8+125*X^6*Y^2+25*X^4*Y^4+5*X^2*Y^6+Y^8)
=(5*X^2-Y^2)*(25*X^4 - 25*X^3*Y + 15*X^2*Y^2 - 5*X*Y^3 + Y^4)*(25*X^4 + 25*X^3*Y + 15*X^2*Y^2 + 5*X*Y^3 + Y^4)
ずいう぀の積の圢に䜜り倉えられる。


そこで
A^7+B^7=(A+B)*(A^6-A^5*B+A^4*B^2-A^3*B^3+A^2*B^4-A*B^5+B^6)
ではあるがA,Bを適圓に眮き盎すこずで
぀の積で䜜った圢に盎しおほしい。

匕甚しお返信線集・削陀(未線集)

4぀だず簡単なので「ちょうど3぀の因数」ずいうこずですよね
それならば䟋えば
A=X^3+1, B=Y^3-1 ずすれば
A^7+B^7=(X^3+1)^7+(Y^3-1)^7
=(X+Y)(X^2-XY+Y^2)
{(X^18-X^15Y^3+X^12Y^6-X^9Y^9+X^6Y^12-X^3Y^15+Y^18)
+7(X^15-X^12Y^3+X^9Y^6-X^6Y^9+X^3Y^12-Y^15)
+21(X^12-X^9Y^3+X^6Y^6-X^3Y^9+Y^12)+35(X^9-X^6Y^3+X^3Y^6-Y^9)
+35(X^6-X^3Y^3+Y^6)+21(X^3-Y^3)+7}

匕甚しお返信線集・削陀(未線集)

なるほど
この発想でもいけるのか。
党く関係ありたせんが䞊の第項目を曞き盎すず
(X^18+Y^18) - X^3*Y^3*(X^12+Y^12)+X^6*Y^6*(X^6+Y^6)-X^9*Y^9 +
7*{ (X^15-Y^15) -X^3*Y^3*(X^9-Y^9) +X^6*Y^6*(X^3-Y^3)} +
21*{(X^12+Y^12) -X^3*Y^3*(X^6+Y^6) +(X^3-Y^3)+X^6*Y^6} +
35*{ (X^9-Y^9) -X^3*Y^3*(X^3-Y^3) +(X^6+Y^6)-X^3*Y^3} +
7

なおこちらが甚意しおいたのが
A=7*X^2,B=Y^2ず眮いお出来る
823543*X^14+Y^14=(7*X^2+Y^2)*(117649*X^12 - 16807*Y^2*X^10 + 2401*Y^4*X^8 - 343*Y^6*X^6 + 49*Y^8*X^4 - 7*Y^10*X^2 + Y^12)
=(7*X^2+Y^2)
* (343*X^6 - 343*Y*X^5 + 147*Y^2*X^4 - 49*Y^3*X^3 + 21*Y^4*X^2 - 7*Y^5*X + Y^6)
         * (343*X^6 + 343*Y*X^5 + 147*Y^2*X^4 + 49*Y^3*X^3 + 21*Y^4*X^2 + 7*Y^5*X + Y^6)
なる匏でした。



ずなんかきれい。

匕甚しお返信線集・削陀(線集枈: 2023幎09月01日 05:37)

[続]分床儀に郜合の良い䞉角圢

再び申し蚳ないのですが
この正倚角圢をどんどん倧きくしお行った時、どうしたらいいのか迷っおいるので
次の問題を考えお頂きたい。

æ­£1000角圢の図圢があるずする。
この任意の頂点3か所を遞んで䜜られた䞉角圢の内角が党郚敎数角ずなる
頂点3か所の遞び方(組合せ)は党郚で䜕通りあるかを求めお欲しい。
䜆し頂点には固有の番号が振り圓おられおいるものずしたす。
できたら
æ­£2024角圢,æ­£2345角圢でも求めお欲しい。

匕甚しお返信線集・削陀(線集枈: 2023幎08月25日 08:11)

æ­£1000角圢
1000=2^3×5^3、180÷(2^2×5)=9ず2×5^2=50は互いに玠なので
敎数角になるためには頂点を50n個(円呚角9n°)単䜍で䜿甚しなければならない。
∎(1000÷50)C3×50=57000通り

æ­£2024角圢
2024=2^3×11×23、180÷2^2=45ず2×11×23=506は互いに玠なので
敎数角になるためには頂点を506n個(円呚角45n°)単䜍で䜿甚しなければならない。
∎(2024÷506)C3×506=2024通り

æ­£2345角圢
2345=5×7×67、180÷5=36ず7×67=469は互いに玠なので
敎数角になるためには頂点を469n個(円呚角36n°)単䜍で䜿甚しなければならない。
∎(2345÷469)C3×469=4690通り

぀たり正N角圢の堎合は
g=gcd(N,180)ずしおgC3×(N/g)通りただしg3のずき0通り
ずいうこずですね。

匕甚しお返信線集・削陀(線集枈: 2023幎08月25日 09:26)

正角圢に぀いお、蟺に察する円呚角は、/なので、個のたずたりごずに敎数角ずなる。
よっお、自然数、、に察しお、 から、 
回転させお重なる解は同䞀芖しお、手䜜業で解を求めるず、通り
よっお、䞉角圢の遞び方は、×通りずなる。
らすかるさんの結果ず䞀臎しお、安心したした

匕甚しお返信線集・削陀(未線集)

分床儀に郜合良い䞉角圢䜜り

䞀般に正n角圢があり
その任意の盞異なる3頂点A,B,Cを遞んで䞉角圢ABCを぀くるずき
内角のすべおが敎数床(1°の敎数倍)ずなるような䞉角圢である
こずが起こる3点の遞び方はそれぞれ䜕通りあるか
(1)n=14
(2)n=15
(3)n=16

匕甚しお返信線集・削陀(未線集)

(2)は
æ­£15角圢のどの3頂点を遞んでも、その3点で䜜られる䞉角圢のすべおの内角は敎数床になるから
15C3=455通り
ずなるような気がしたすが、
もし私の勘違いでしたらご指摘䞋さい。

匕甚しお返信線集・削陀(未線集)

を蚈算しおみたした。点の遞び方が問題なので、合同や裏返しで重なる䞉角圢も異なるず芋なしおよい。

 正角圢の蟺に察する円呚角は、°で、これらを組み合わせお䞉角圢を䜜るこずになる。
よっお、個の頂点から぀の頂点を遞ぶ堎合の数は、153通り。
圓初、考え違いをしおいるこずに気づき、修正したした。

 正角圢の蟺に察する円呚角は、°で、これらを組み合わせお䞉角圢を䜜るには、
、、の組み分けで䞉角圢が䜜られる。
 よっお、求める堎合の数は、×通り

匕甚しお返信線集・削陀(線集枈: 2023幎08月24日 16:23)

(2)は455(通り)の組合せで、お二人ずも正解です。
(3)は私の解ず管理人さんずは異なっおいたす。
できたらその48通りは具䜓的に頂点の郚分を{1,2,3,,16}
ずするずき、どの頂点の3぀を遞んでいるのかを瀺しおくれたせんか。

匕甚しお返信線集・削陀(未線集)

党く自信はありたせんが。
頂点を、、・・・、ずした堎合、が衚す䞉角圢ずしお
ずか10ずか、・・・で、∠∠°、∠°
ずなりたす。

匕甚しお返信線集・削陀(未線集)

倖接円を考えたずきに、任意の 2 頂点間にできる䞭心角が偶数床になればいいず考えたす。

(1)
360/14 を敎数倍しお偶数を䜜るには 7 の倍数を掛けるしかなく、正の 7 の倍数 3 ぀合蚈で 14 にはできたせん。
よっお 0 通り。

(2)
360/15 を敎数倍しお偶数を䜜るには任意の敎数でよく、結局 A,B,C を重耇しないように任意に遞べばいいです。
よっお 15P3 = 2730 通り。

(3)
360/16 を敎数倍しお偶数を䜜るには 4 の倍数を掛けるしかなく、正の 4 の倍数 3 ぀合蚈で 16 になるのは 4,4,8 ずいう組み合わせのみ。
぀たり盎角二等蟺䞉角圢を䜜るしかありたせん。
A が盎角なものが 16*2 = 32 通り、B ず C に぀いおも同じ個数あるので、党郚で 32*3 = 96 通り


##「3頂点を遞んで䞉角圢を぀くる」のではなく、「3頂点A,B,Cを遞んで䞉角圢ABCを぀くる」問題ですから、点の名前が入れ替われば別物ずすべきだず思いたす。

匕甚しお返信線集・削陀(未線集)

(1)は90°の角床は䜜れるが1,2,9の頂点など)他の内角は敎数になれなく、結局0(通り)
(2)は出題の時3点を䜕気にA,B,Cず蚀っおしたったのでDD++さんの解釈が起こったが(こう質問するずDD++さんが正しい。)
こちらが思っおいたのは、15個の頂点の3぀の組合せが幟぀取れるか
の぀もりで考えおいたので15C3=455(通り)でお願いしおおきたす。
(3)は盎角二等蟺䞉角圢なら、条件を満たすので、これも頂点1,2,3,,16
からの3぀の頂点の遞び方は各頂点に90°の郚分がくる堎合の16通り)
ずいう予定でした。

æ­£n倚角圢ず、その頂点の任意の3点を結んで䜜る䞉角圢の3぀の内角がどれも敎数角ずなれるのが
n=5 -->各内角は36°の敎数倍
n=6 -->各内角は30°の敎数倍
n=9 -->各内角は20°の敎数倍
n=10 -->各内角は18°の敎数倍
n=12 -->各内角は15°の敎数倍
n=15 -->各内角は12°の敎数倍
n=18 -->各内角は10°の敎数倍
n=20 -->各内角は9°の敎数倍
n=30 -->各内角は6°の敎数倍
n=36 -->各内角は5°の敎数倍
が起こせるんですね。
蚈算をしお初めお気付けたした。

匕甚しお返信線集・削陀(未線集)

指数和

aずbは互いに玠な正の敎数、kは2以䞊の敎数、nはabの倍数である正の敎数ずしたす。
以䞋のΣはΣ[x[1]=1]Σ[x[2]=1n] Σ[x[k]=1n]の意味ずしたす。
f(m)=e^(2πi x[1]x[2]
x[k]/m)ずしたす(i=√-1)。
Σf(a)*Σf(b)/Σf(n)の倀を求めお䞋さい。

匕甚しお返信線集・削陀(未線集)

勘違いだったらすみたせんが、これ分母が 0 になりたせんか

匕甚しお返信線集・削陀(未線集)

結合法則を満たすずは

結合法則ずは
ある挔算∘に察し
(a∘b)∘c=a∘(b∘c)
がい぀も成り立぀こずを指す。

そこで今集合Mの挔算を
Mの二぀の元に察しMの元䞀぀を察応させる芏則
f:M×M→Mぞの写像
f(a,b)=a∘b  (∀(a,b)∈M×M, ∃a∘b∈M )
で定矩するこずにする。

さおこの時
(1)Mの元が2個である時
M×Mの元は2^2=4個でM×MよりMぞの写像はM×Mの各元に察し
2通りある行き先を指定するので、党郚で2^(2^2)=16(通りの写像が
考えられる。
ではこの䞭で結合法則を満たす写像は䜕通りあるか

(2)Mの元が3個である時
党郚で3^(3^2)=19683(通り)の写像の䞭で
結合法則を満たす写像は䜕通りあるか

(3)Mの元が4個である時
党郚で4^(4^2)=4294967296(通り)の写像の䞭で
結合法則を満たす写像は䜕通りあるか

匕甚しお返信線集・削陀(未線集)

(1)は、通りずなりたした。

匕甚しお返信線集・削陀(未線集)

ある本を読んでいるずき、この写像ず結合法則の組合せに぀いおの蚘述を読んで
実際どんな写像挔算)が条件を満たすのかを知りたくなり、Mの芁玠が2぀の堎合に
党郚(16通り)を党おチェックしたら、管理人さんず同様に8通りであるこずを実隓から
芋぀けられたした。
でもこれを前もっおわかるこずはどうしおも芋぀けられなく、次のMの芁玠が3個の堎合は
党郚で19683通りもあるので、䜕ずかコンピュヌタを利甚しおカりントしない限り分からない
ず感じそのプログラムをどう蚭蚈すれば可胜なのかず、あれこれ詊行錯誀を繰り返しお組み䞊げお
いきたした。䜕日も組み方が分からず悪戊苊闘の連続でした。
やっずこれで求たるのではないかず思われるプログラムで蚈算した結果が113通りでした。
Mの芁玠が4なら3492通りになりたした。(結果が出るたで随分時間がかかりたした。
Mの芁玠が2の堎合に范べ、その比率が極端に小さくなったのでこの結果は自信がありたせんでした。
この僅かの8,113,3492を䟋のOEISで怜玢するず
A023814がヒットしたした。
でもこのサむトでの説明文では䜕も結合法則なる蚘述はなく、数は䞀臎するもこれが求める数を
瀺すものかいたいち自信がありたせん。
䜕方かこの数を瀺す正しい数倀を芋぀けお貰いたいのですが

もしこの数倀が正しいなら結合法則が成り立぀ずはずおも珍しい珟象であるず認識しないず
いけないものだず思える。

匕甚しお返信線集・削陀(未線集)

GAIさん

>この僅かの8,113,3492を䟋のOEISで怜玢するず
>A023814がヒットしたした。
>でもこのサむトでの説明文では䜕も結合法則なる蚘述はなく、数は䞀臎するもこれが求める数を
>瀺すものかいたいち自信がありたせん。


英語の意味を怜玢するず、

associative : 〔挔算などが〕結合的な、結合埋法則を満たす

binary operation : 二項挔算

らしいので、

" Number of associative binary operations on an n-set "

は

「芁玠数nの集合における結合法則を満たす二項挔算の数」

ずなっおGAIさんが知りたいものそのものではないでしょうか

匕甚しお返信線集・削陀(未線集)

亀通敎理 その

前回たでは、合成数では、蚈算䞊では正しいのですが、理屈では、右蟺ず巊蟺の倍数が等しくなりたせんでした。
そこで、解決できたした。

等比玚数の和の公匏より
N^s-1=(N-1){1+N+N^2+n^3+・・・・+N^(s-2)+N^(s-1)}------(1)
さお、s=4のずき、
N(N-1)(N-2)(N-3)=(N^4-N)-6(N^3-N)+11(N^2-N)-----(2)
=N(N^3-1)-6N(N^2-1)+11N(N-1)
であるから、(1)より、
=N(N^3-1)-6N(N^2-1)+11N(N-1)
=N{(N-1)(1+N+N^2)-6(N-1)(1+N)+11(N-1)}
=N(N-1){(1+N+N^2)-6(1+N)+11}
=N(N-1){(1+N+N^2-6-6N+11}
=N(N-1){(N^2-5N+6}
=N(N-1)(N-2)(N-3)
これより、(2)の右蟺は巊蟺に等しい。

さお、s=6のずき、
N(N-1)(N-2)(N-3)(N-4)(N-5)=(N^6-N)-15(N^5-N)+85(N^4-N)-225(N^3-N)+274(N^2-N)------(3)
であるから、(1)より、
=N(N^5-1)-15N(N^4-1)+85N(N^3-1)-225N(N^2-1)+274N(N-1)
=N(N-1){(1+N+N^2+N^3+N^4)-15(1+N+N^2+N^3)+85(1+N+N^2)-225(N+1)+274}
=N^4-14N^3+71N^2-154N+120
=N(N-1)(N-2)(N-3)(N-4)(N-5)
これより、(3)の右蟺は巊蟺に等しい。

さお、s=8のずき、
N(N-1)(N-2)(N-3)(N-4)(N-5)(N-6)(N-7)=(N^8-N)-28(N^7-N)+322(N^6-N)-1960(N^5-N)+6769(N^4-N)-13132(N^3-N)+13068(N^2-N)------(4)
であるから、(1)より、
=N{(N^7-1)-28(N^6-1)+322(N^5-1)-1960(N^4-1)+6769(N^3-1)-13132(N^2-1)+13068(N-1)}
=N(N-1){(1+N+N^2+N^3+N^4+N^5+N^6)-28(1+N+N^2+N^3+N^4+N^5)+322(1+N+N^2+N^3+N^4)-1960(1+N+N^2+N^3)+6769(1+N+N^2)-13132(1+N)+13068}
=N^6-27N^5+295N^4-1665N^3+5104N^2-8028N+5040
=N(N-1)(N-2)(N-3)(N-4)(N-5)(N-6)(N-7)
これより、(4)の右蟺は巊蟺に等しい。

よっおSが自然数のずき
N(N-1)(N-2)(N-3)・・・・{N-(S-1)}=(N^S-N)-a1{N^(S-1)-N}+a2{N^(S-2)-N}・・・・-a(s-2)(N^3-N)+a(s-1)(N^2-N)
は、sが合成数でも成り立぀。

匕甚しお返信線集・削陀(線集枈: 2023幎08月19日 08:42)

぀たり、なぜ理屈が間違うかずいうず、右蟺は、sが合成数の堎合(N^t-N){ただし、tはs以䞋のすべおの自然数}の係数がsの倍数にならないずいうのが、前回の結論でした。
ただし、(N^t-N)がtの倍数ずしたら、成り立぀ようです。
(%i1) factor(4-6*3+11*2);
(%o1) 2^3=4X2
(%i2) factor(6-15*5+85*4-225*3+274*2);
%o2) 2^4 3^2=6^2x2^2
(%i3) factor(8-28*7+322*6-1960*5+6769*4-13132*3+13068*2);
(%o3) 2^7 3^3 5=8x2^4x3^3x5

線集枈み

匕甚しお返信線集・削陀(線集枈: 2023幎08月19日 09:11)

うんざりはちべえさん、おはようございたす。

よっおSが自然数のずき
N(N-1)(N-2)(N-3)・・・・{N-(S-1)}=(N^S-N)-a1{N^(S-1)-N}+a2{N^(S-2)-N}・・・・-a(s-2)(N^3-N)+a(s-1)(N^2-N)
は、sが合成数でも成り立぀。

これは前回自分で蚌明されたしたよね。No.1340の投皿です。

「因みに、䜕次でも-abcdN+N-(a+b+c+d)N+(ab+bc+cd+ac+ad+bd)N-(abc+abd+acd+bcd)Nの郚分はNの項になり、解ず係数の関係ず同じで±が亀互になり、必ず ±(a - 1) (b - 1) (c - 1) (d - 1) ず因数分解でき、より、
N(N-1)(N-2)(N-3)・・・・{N-(S-1)}=(N^S-N)-a1{N^(S-1)-N}+a2{N^(S-2)-N}・・・・-a(s-2)(N^3-N)+a(s-1)(N^2-N)の圢に出来るのですね。」No.1342より

このには合成数ずか玠数ずか制限がないので、䞀般ので成り立぀ずいう事ですね。

぀たり、なぜ理屈が間違うかずいうず、右蟺は、sが合成数の堎合(N^t-N){ただし、tはs以䞋のすべおの自然数}の係数がsの倍数にならないずいうのが、前回の結論でした。

理屈は間違っおいないず思いたす。䞀応、前回のものを挙げおおきたすね。

等匏は成り立぀ので、巊蟺の倍数ず右蟺の倍数は等しいのです。
ずころが、合成数のずき、巊蟺ず右蟺が䞀臎しないずいう理屈がおかしいのです。

この右蟺が×だけで぀ながった匏ならおかしいですが、(N^4-N)-6(N^3-N)+11(N^2-N)は和ず差で぀ながっおいるのでおかしくありたせん。確か、NHKの番組でも掛け算は簡単ですが足し算は難しいずいうような話をやっおいたしたよね。それず同じ事です。

ただし、(N^t-N)がtの倍数ずしたら、成り立぀ようです。
(%i1) factor(4-6*3+11*2);
(%o1) 2^3=4X2
(%i2) factor(6-15*5+85*4-225*3+274*2);
%o2) 2^4 3^2=6^2x2^2
(%i3) factor(8-28*7+322*6-1960*5+6769*4-13132*3+13068*2);
(%o3) 2^7 3^3 5=8x2^4x3^3x5

これは、DD++さんが発芋した、N^561-Nはの倍数ずいう合成数561でやっおみお䞋さい。倚分、ダメですよ。倚分の所は蚌明しおありたすが、今回は省略したす。

匕甚しお返信線集・削陀(未線集)

壊れた扉様、こんにちは。

s=561ですか
N(N-1)(N-2)(N-3)・・・(N-560)=(N^561-N)-a1(N^560-N)+a2(N^559-N)・・・・a561(N^2-N)
ずおもそんな蚈算は無理です。

匕甚しお返信線集・削陀(未線集)

ええ、私も以前はあきらめおいたしたが、DD++さんは凄いですよね。

https://www.wolframalpha.com/input?i=Table%5B%28N%5E561-N%29mod561%2C%7BN%2C2%2C30%7D%5D&lang=ja

pythonずかず違っお桁違いの蚈算力なんですね。因みに、今回のに䜿えるかどうかは党く考えおいたせん。

匕甚しお返信線集・削陀(未線集)

DD++様の蚈算は、
N=2〜30においお(N^561ヌN) mod 561を求めるものです。

匕甚しお返信線集・削陀(線集枈: 2023幎08月19日 14:36)

うんざりはちべえさん、こんにちは。

よく芋るず、以前ずは違う法則ですね。

぀たり、なぜ理屈が間違うかずいうず、右蟺は、sが合成数の堎合(N^t-N){ただし、tはs以䞋のすべおの自然数}の係数がsの倍数にならないずいうのが、前回の結論でした。
ただし、(N^t-N)がtの倍数ずしたら、成り立぀ようです。
(%i1) factor(4-6*3+11*2);
(%o1) 2^3=4X2
(%i2) factor(6-15*5+85*4-225*3+274*2);
%o2) 2^4 3^2=6^2x2^2
(%i3) factor(8-28*7+322*6-1960*5+6769*4-13132*3+13068*2);
(%o3) 2^7 3^3 5=8x2^4x3^3x5

s=4のずき、
N(N-1)(N-2)(N-3)=(N^4-N)-6(N^3-N)+11(N^2-N)-----(2)
=N(N^3-1)-6N(N^2-1)+11N(N-1)

s=6のずき、
N(N-1)(N-2)(N-3)(N-4)(N-5)=(N^6-N)-15(N^5-N)+85(N^4-N)-225(N^3-N)+274(N^2-N)------(3)
であるから、(1)より、
=N(N^5-1)-15N(N^4-1)+85N(N^3-1)-225N(N^2-1)+274N(N-1)

s=8のずき、
N(N-1)(N-2)(N-3)(N-4)(N-5)(N-6)(N-7)=(N^8-N)-28(N^7-N)+322(N^6-N)-1960(N^5-N)+6769(N^4-N)-13132(N^3-N)+13068(N^2-N)------(4)
であるから、(1)より、
=N{(N^7-1)-28(N^6-1)+322(N^5-1)-1960(N^4-1)+6769(N^3-1)-13132(N^2-1)+13068(N-1)}

以前ず違っお、係数に指数をかけお総和を取っおいるのですね。

匕甚しお返信線集・削陀(未線集)

゚レベヌタ蚭眮の蚭蚈

9階建おのビルがあり、ここにどこかの3ヵ所の階にしか止たらない12台の゚レベヌタを蚭眮する
こずをする。
各゚レベヌタがどこの階に止たるのかを䞊手く組み合わせるず、どの階にいおも他の階に行ける
゚レベヌタが必ずあっお、各階には4぀の゚レベヌタが運行しおいる状態になる様な蚭蚈が可胜
ずなりたす。
そこでその蚭蚈に挑戊しおみお䞋さい。
゚レベヌタE1,E2,,E12が止たる階を3぀それぞれ指定しおみおください。

匕甚しお返信線集・削陀(未線集)

この゚レベヌタの問題よりもさらに厳しい条件の問題が
「私の備忘録 > 拡匵カヌクマン問題」
の䞭にプレカヌクマン問題ずしお茉っおいたす。
プレカヌクマン問題の解は、より条件の緩いこの゚レベヌタ問題の解ずしおも適したす。

匕甚しお返信線集・削陀(未線集)

゚レベヌタ問題ずカヌクマンの女生埒問題は連動しおいるんですね。
䞀般に゚レベヌタの総数を(2*n+1)*(3*n+1)で各゚レベヌタはどこかの
階の3か所で皌働するように動くずき、どの階からも他の階に行けるように
なるためのビルの高さの最倧階数は6*n+3ずなる。
これをf((2*n+1)*(3*n+1),3)=6*n+3 で衚しおおく。
これよりn=1,2,3,4で圓おはめるず
n=1で12台の゚レベヌタでは9Fたでのビル(出題の問題)
n=2で35台の゚レベヌタでは15Fたでのビル(カヌクマンの女生埒の解を利甚できる。)
n=3で70台の゚レベヌタでは21Fたでのビル
n=4で117台の゚レベヌタでは27Fたでのビル
に察しお蚭蚈できる。

たた他にも
f(s^2+s,s)=s^2のようなものも,
゚レベヌタ総数がs^2+sでビルの高さがs^2階たでの時は
各゚レベヌタがどれもs個の階だけしか止たらない動きをすれば
どんな階からでも他の階ぞ行ける゚レベヌタを運行できる。
これよりs=2,3,4,5,6,7から
f(6,2)=4
f(12,3)=9
f(20,4)=16
f(30,5)=25
f(42,6)=36
f(56,7)=49

以前私の備忘録 > 拡匵カヌクマン問題で
りらひいさんが投皿されおいた

方陣[※]
0 1 2 3 4 5 6
7 8 9 10 11 12 13
14 15 16 17 18 19 20
21 22 23 24 25 26 27
28 29 30 31 32 33 34
35 36 37 38 39 40 41
42 43 44 45 46 47 48

魔方陣[1]
3 34 9 40 15 46 21
28 10 41 16 47 22 4
11 35 17 48 23 5 29
36 18 42 24 6 30 12
19 43 25 0 31 13 37
44 26 1 32 7 38 20
27 2 33 8 39 14 45

魔方陣[2]
38 6 16 33 43 11 21
0 17 34 44 12 22 39
18 28 45 13 23 40 1
29 46 7 24 41 2 19
47 8 25 35 3 20 30
9 26 36 4 14 31 48
27 37 5 15 32 42 10

魔方陣[3]
17 13 2 47 36 32 21
7 3 48 37 33 22 18
4 42 38 34 23 19 8
43 39 28 24 20 9 5
40 29 25 14 10 6 44
30 26 15 11 0 45 41
27 16 12 1 46 35 31

の各方陣の各行、各列を利甚させお貰うずこのf(56,7)=49
のモデルを䜜るこずが出来たした。

远䌞
このモデルを぀くっおおけば
f(s^2-s+1,s)=s^2-s+1
でのs=8
即ちf(57,8)=57でのモデル
党郚で57台の゚レベヌタを57階建おのビルに蚭眮し
各゚レベヌタがどこかの階の8ヵ所を皌働するように䞊手く組み合わせおおけば
どの階からも任意の階ぞ運行しおいる゚レベヌタが存圚しおいる蚭蚈が難なく出来る。
(䞊のモデルの行ず列を入れ替えお、残りの゚レベヌタE50E57に察する停止の8ヵ所の階を理詰めで決定。)

したがっおりらひいさんのs:玠数での
f(s^2+s,s)=s^2のモデルを構成できる汎甚的構成方法を䜿えば
f((s+1)^2-(s+1)+1,s+1)=(s+1)^2-(s+1)+1
即ち
f(13,4)=13
f(31,6)=31
f(57,8)=57
f(133,12)=133
f(183,14)=183
f(307,18)=307
f(381,20)=381
f(553,24)=553
f(871,30)=871
f(993,32)=993
f(1407,38)=1407
f(1723,42)=1723
f(1893,44)=1893
f(2257,48)=2257

のモデルも順次䜜っおいける。
こんなものを詊行錯誀で䜜るこずは、殆んど望めない。)

匕甚しお返信線集・削陀(線集枈: 2023幎08月19日 06:22)
合蚈1737件 (投皿284, 返信1453)

ロケットBBS

Page Top